Desmopressin reduces urine volume in which of the following condition?
- A. Central (neurogenic) diabetes insipidus
- B. Renal (nephrogenic) diabetes insipidus
- C. Both (a) and (b)
- D. Neither (a) nor (b)
Correct Answer: A
Rationale: Desmopressin reduces urine volume by mimicking the effects of antidiuretic hormone (ADH). In central diabetes insipidus, there is a deficiency of ADH production, so desmopressin can help by supplementing ADH. In renal diabetes insipidus, the kidneys do not respond to ADH, so desmopressin would not be effective. Therefore, the correct answer is A. Choice B is incorrect because desmopressin does not address the underlying issue of kidney insensitivity to ADH. Choice C is incorrect because desmopressin is effective only in central diabetes insipidus. Choice D is incorrect because desmopressin is effective in central diabetes insipidus.
You may also like to solve these questions
Superinfections are more common with:
- A. Use of narrow spectrum antibiotics
- B. Short courses of antibiotics
- C. Use of antibiotics that are completely absorbed from the small intestines
- D. Use of antibiotic combinations covering both gram positive and gram negative bacteria
Correct Answer: D
Rationale: The correct answer is D because using antibiotic combinations covering both gram positive and gram negative bacteria can disrupt the balance of normal flora in the body, leading to superinfections. Gram positive and gram negative bacteria have different susceptibility patterns, so using a combination of antibiotics increases the likelihood of killing off a wider range of bacteria, including the beneficial ones. This disruption paves the way for opportunistic pathogens to overgrow and cause superinfections. Choices A, B, and C are incorrect because narrow spectrum antibiotics target specific bacteria, short courses of antibiotics reduce the risk of resistance and overgrowth, and antibiotics completely absorbed from the small intestines have minimal impact on gut flora compared to systemic antibiotics.
A 22-year-old woman presents to the clinic with a complaint of severe menstrual cramps. The nurse suspects dysmenorrhea. Which of the following is the most likely cause of dysmenorrhea?
- A. Uterine contractions.
- B. Endometriosis.
- C. Ovarian cysts.
- D. Polycystic ovary syndrome (PCOS).
Correct Answer: C
Rationale: The correct answer is C: Ovarian cysts. Ovarian cysts can cause dysmenorrhea by disrupting the normal menstrual cycle and causing pain. Endometriosis (B) is another common cause of dysmenorrhea, but it involves the abnormal growth of endometrial tissue outside the uterus. Uterine contractions (A) are a normal part of menstruation and may cause cramping but are not the primary cause of dysmenorrhea. PCOS (D) is a hormonal disorder that can cause irregular periods and pain, but it is not the most likely cause of dysmenorrhea in this case.
Mrs. Miller has a diagnosis of acute pyelonephritis. To prevent renal damage her plan of care should include
- A. restricting fluid intake until symptoms are under control
- B. reducing protein intake
- C. understanding the necessity of continuing drug therapy indefinitely
- D. preventing viral infections
Correct Answer: C
Rationale: Correct Answer: C
Rationale:
1. Acute pyelonephritis requires antibiotic therapy to eliminate the infection and prevent renal damage.
2. Continuing drug therapy indefinitely ensures complete eradication of the infection.
3. Stopping drug therapy prematurely can lead to recurrence and potential renal complications.
4. Understanding the necessity of drug therapy is crucial for Mrs. Miller's long-term renal health.
Summary:
A: Incorrect. Restricting fluid intake can worsen the infection and hinder recovery.
B: Incorrect. Protein intake is not directly related to the treatment of acute pyelonephritis.
D: Incorrect. Preventing viral infections is not directly relevant to the treatment of acute pyelonephritis.
A 50-year-old woman calls the clinic because she has noticed some changes in her body and breasts and wonders if these changes could be attributable to the hormone replacement therapy (HRT) she started 3 months earlier. The nurse should tell her:
- A. HRT is at such a low dose that side effects are very unusual.
- B. HRT has several side effects, including fluid retention, breast tenderness, and vaginal bleeding.
- C. Vaginal bleeding with HRT is very unusual; I suggest you come into the clinic immediately to have this evaluated.
- D. It sounds as if your dose of estrogen is too high; I think you may need to decrease the amount you are taking and then call back in a week.
Correct Answer: B
Rationale: The correct answer is B because hormone replacement therapy (HRT) commonly causes side effects such as fluid retention, breast tenderness, and vaginal bleeding. This information is important for the nurse to communicate to the patient, as it aligns with the symptoms the woman is experiencing. Choice A is incorrect as even low doses of HRT can still result in side effects. Choice C is incorrect because vaginal bleeding with HRT is not very unusual and does not necessitate immediate evaluation unless it is severe. Choice D is incorrect as adjusting the dose without proper evaluation could be dangerous and is not the appropriate first step in addressing the patient's concerns.
A 75 year old male with BPH has continues to have urinary symptoms of * prostatic obstruction after trial of alpha blocking drug tamsulosin . Dutasteride is added to his therapy,he is also taking hydrochlorothiazide sildenafil and
- A. Which of the following medication could have an interaction with dutasteride?
- B. Tamsulosin
- C. Testosterone
- D. Seldinal
Correct Answer: D
Rationale: The correct answer is D: Sildenafil. Dutasteride and sildenafil are both metabolized by the same enzymes in the liver, specifically CYP3A4 and CYP2C9. When taken together, sildenafil can increase the blood levels of dutasteride, leading to a higher risk of side effects. Tamsulosin (choice B) is commonly used in combination with dutasteride for BPH without significant interactions. Testosterone (choice C) is not typically used in BPH treatment and does not interact significantly with dutasteride. Therefore, the potential interaction between dutasteride and sildenafil is the reason why choice D is correct.